Calendrier de l’Avent - Page 8 — Les-mathematiques.net The most powerful custom community solution in the world

Calendrier de l’Avent

1234568

Réponses

  • Pas sûr que ceci soit vrai, en prenant J très lacunaire. Ce que je montre c'est plutôt 
    $$\underset{x \to +\infty}{\limsup} \dfrac{f_J(x)}{\frac{e^x}{\sqrt x}}\geq \frac{1}{\sqrt{2\pi}}.$$

    Ceci est cependant suffisant pour l'exercice d'étanche.
  • Puisque que personne ne s’est présenté pour le J24, je propose de résoudre cette question resté ouverte sur le forum :smile:
    https://les-mathematiques.net/vanilla/index.php?p=/discussion/239821
    Je suis donc je pense 
  • Modifié (December 2022)
    Namiswan
    En fait j'ai perfectionné un peu ta démonstration en minorant $\displaystyle\sum_{k=n}^{n+p}\dfrac{n^k}{k!}$ par une suite équivalente à $(p+1)\dfrac{e^n}{\sqrt{2\pi n}}$.
    On peut choisir $p$ arbitrairement grand.

    Edit : merci à Calli d'avoir remarqué que cela ne marche que si $J$ contient des suites arbitrairement longues d'entiers consécutifs.
  • Modifié (December 2022)
    @jandri : il faudrait des détails, mais de loin je ne crois pas trop à ton énoncé

    @Quentino37 : si tu veux je propose un truc.

  • Modifié (December 2022)
    @jandri : On dirait que tu supposes que $J$ contient des suites arbitrairement longues d'entiers consécutifs.

    Il me semble qu'une comparaison série-intégrale donne $\displaystyle \sum_{n=0}^\infty \frac{x^{n^2}}{(n^2)!} = O\!\left(\frac{e^x}{\sqrt{x}}\right)$ quand $x\to\infty$.
  • Modifié (December 2022)
    Namiswan a dit :
    @jandri : il faudrait des détails, mais de loin je ne crois pas trop à ton énoncé
    @Quentino37 : si tu veux je propose un truc.
    Si tu veux ! 
    Je suis donc je pense 
  • Modifié (December 2022)
    Ok, voici. On approche de la fin, donc je monte un peu le niveau par rapport à mes précédentes questions.
    C'est de la récup mais à ma connaissance, aucune solution n'est disponible à l'heure actuelle.

    Jour 24


    Soit $f:\R\to\R$ une fonction périodique et lipschitzienne telle que $f(0)=0$. Peut-on connaître la nature de la série $$ \sum_{n\geq 1}\frac{1}{n^{1+f(n)}}\quad ?$$
  • Modifié (December 2022)
    Calli a dit :
    @jandri. On dirait que tu supposes que $J$ contient des suites arbitrairement longues d'entiers consécutifs.
    Merci à Calli (et à Namiswan), c'est exactement ce que j'ai supposé (par erreur puisque ce n'est pas dans les hypothèses sur $J$).
    Dans le cas général ma "démonstration" tombe en défaut !
  • J 24 en réfléchissant au brouillon j’ai l’impression ça diverge, si je ne me suis pas trompé.
  • C'est effectivement de là que j'ai tiré la question :)
    (je suis l'auteur de cette partie)
  • Bravo @Namiswan , le problème n'a pas tenu longtemps !

    Je donne un autre exercice rigolo pour les intéressés :

    Donner un équivalent de $\sum_{n=0}^{+\infty} \frac{x^n}{(n!)^2}$ quand $x \to +\infty$. En fait ça peut se faire pour n'importe quel exposant à la place de $2$. ;)
  • Avec l'exposant $2$, on peut utiliser les intégrales de Wallis pour écrire la somme sous forme intégrale puis utiliser une méthode de Laplace.
    Pour les autres cas, c'est plus technique :)
  • Cidrolin pose toujours des questions inédites et difficiles. J’ai hâte de voir son exercice. 
  • Modifié (December 2022)
    Poirot a dit :
    Je donne un autre exercice rigolo pour les intéressés :
    Donner un équivalent de $\sum_{n=0}^{+\infty} \frac{x^n}{(n!)^2}$ quand $x \to +\infty$. En fait ça peut se faire pour n'importe quel exposant à la place de $2$. ;)
    La somme de cette série entière  est $I_0(2\sqrt{x})$ où $I_0$ est la fonction de Bessel de première espèce. 
    Et $I_0(x) \sim \frac{e^x}{\sqrt {2\pi x}}$ quand $x$ tend vers $+\infty$ 
  • Modifié (December 2022)

    Jour 25


  • Modifié (December 2022)
    Réponse à l'énigme du jour 24 : 

    Montrons que la série diverge dans tous les cas. Soit $T>0$ une période de $f$.

     

    Cas 1 : $T\in \mathbb{Q}$.

    Comme $nT$ est une période de $f$ pour tout $n\in \mathbb{N}^*$, on peut supposer que $T\in \mathbb{N}^*$. Alors on a \[\sum_{n=1}^\infty  \frac{1}{n^{1+f(n)}} \geqslant  \sum_{k=1}^\infty  \frac{1}{(kT)^{1+f(kT)} } = \sum_{k=1}^\infty  \frac{1}{kT} = +\infty .\]


    Cas 2 : $T\not\in \mathbb{Q}$.

    Pour tout $x\in \mathbb{R}$, notons $m(x)$ le représentant modulo $T$ de $x$ qui appartient à $]- \frac{T}{2} , \frac{T}{2} ]$. Soit $N\in \mathbb{N}^*$ pair. L'ensemble $\{m(i), i\in [\![0,N]\!]\}$ contient $N+1$ points distincts dans $]- \frac{T}{2} , \frac{T}{2} ]$, donc par principe des tiroirs il existe $i,j\in [\![0,N]\!]$ distincts tels que $m(i)$ et $m(j)$ appartiennent au même intervalle $]p \frac{T}{N} , (p+1) \frac{T}{N} ]$ avec $p\in [\![- \frac{N}{2} , \frac{N}{2} -1]\!]$. Ainsi, $|m(i)-m(j)|< \frac{T}{N}$, donc $n_0 :=|i-j|$ vérifie $$n_{0} \in [\![1,N]\!]\quad \text{et}\quad m(n_{0} )\in \left[- \frac{T}{N} , \frac{T}{N} \right]. $$ Sans perte de généralité, on peut supposer que $|m(n_0)| = \min (\{|m(n)|, n\in [\![1,N]\!] \})$. Et supposons par exemple que $m(n_{0} )\in [0, \frac{T}{N} ]$ (le cas $m(n_{0} )\in [- \frac{T}{N},0 ]$ est similaire et ne sera pas détaillé).

    Soit $M\in \mathbb{N}^*$ l'unique entier tel que $\frac{T}{M} < m(n_{0} ) \leqslant  \frac{T}{M-1}$ (*) (on a $M>N$). En réappliquant le principe des tiroirs avec $M$ au lieu de $N$, on trouve qu'il existe $n' \in  [\![1,M]\!]$ tel que $m(n') \in [- \frac{T}{M} , \frac{T}{M} ]$ (**). Et par minimalité de $n_{0}$, on a $n'>n_{0}$. Donc, en soustrayant des inégalités (*) et (**), on a $- \frac{T}{M} - \frac{T}{M-1} \leqslant  m(n' -n_{0} ) \leqslant  \frac{T}{M} - \frac{T}{M}$.

    Ainsi, en notant $n_{1} = n'-n_{0}$, on a \[n_{0} , n_{1} \in  [\![1,M]\!], \quad m(n_{0} ) \in \left[0, \frac{3T}{M} \right] \quad \text{et} \quad m(n_{1} ) \in \left[- \frac{3T}{M} ,0\right].\] Ensuite, définissons la suite d'entiers $(n_{k} )_{k\in \mathbb{N}}$ par : $$\forall k\geqslant 2, \qquad n_{k} = \left\{\begin{array}{ll} n_{k-1} + n_{0}&\text{si } m(n_{k-1})<0\\ n_{k-1} +n_{1}&\text{sinon}\end{array}\right.$$ Par récurrence, elle vérifie : \[\forall k\in \mathbb{N}^*, \qquad n_{k} \in  [\![1,kM]\!] \quad \text{et} \quad m(n_{k} ) \in \left[- \frac{3T}{M} , \frac{3T}{M} \right].\] Informellement, $(n_{k})$ est une suite qui fait modulo $T$ des petits pas vers la gauche ou la droite de manière à toujours rester dans le voisinage de 0 de rayon $\frac{3T}{M}$.

    En outre, en notant $\kappa $ la constante de Lipschitz de $f$, on a : $\forall x\in \mathbb{R}, f(x) =f(m(x))\leqslant  \kappa  |m(x)|$. Donc : $$\begin{eqnarray*} \sum _{n=n_1} ^{\infty } \frac{1}{n^{1+f(n)}} &\geqslant & \sum _{k=1} ^{\infty } \frac{1}{n_{k} ^{1+f(n_{k} )} } \\ &\geqslant & \sum _{k=1} ^{\infty } \frac{1}{n_{k} ^{1+\kappa |m(n_{k} )|}} \\ &\geqslant & \sum _{k=1} ^{\infty } \frac{1}{(kM) ^{1+ \frac{3\kappa T}{M} }} \\&\geqslant & \int_1^{\infty } \frac{1}{(tM) ^{1+ \frac{3\kappa T}{M} }} \, \mathrm{d}t \\ &=& \frac{M}{3\kappa T} \frac{1}{M ^{1+ \frac{3\kappa T}{M} }} \\ &=& \frac{1}{3\kappa T\cdot M ^{ \frac{3\kappa T}{M} }} \\ &\underset{N\rightarrow \infty }{\longrightarrow} & \frac{1}{3\kappa T} \end{eqnarray*}$$ car $N\rightarrow \infty $ implique $M\rightarrow \infty $. Cependant, $n_1 \underset{{N}\rightarrow \infty }{\longrightarrow} \infty $. En effet, sinon on pourrait extraire de $N \mapsto  n_{1}$ une sous-suite constante par rapport à $N$, et en notant $c$ cette constante, on aurait $m(c)=0$ et $c\in \mathbb{N}$, donc $T\in \mathbb{Q}$. Donc on devrait avoir $\sum\limits _{n=n_1} ^{\infty } \frac{1}{n^{1+f(n)}} \underset{N\rightarrow \infty }{\longrightarrow} 0$ si la série convergeait. D'où $\sum\limits_{n=1}^\infty  \frac{1}{n^{1+f(n)}} =+\infty $.


     PS: @Namiswan, dans ton énoncé, tu devrais faire commencer la somme à $n=1$.
  • Jolie preuve, bravo Calli!

    Oui, je corrige la coquille sur l'indice.
  • Bonjour,
    Que faire ?
    Donner la réponse ou donner un indice ou laisser du temps ?
    Amicalement
    Edouard Cidrolin

  • Bonjour Cidrolin'
    Je n'ai pas encore eu le temps de chercher, alors je veux bien encore un dėlai.
    Cordialement, j__j
  • C'est d'accord john_john.
    Cordialement
    E C
  • Bonjour, Cidrolin,
    je rends les armes ! J'ai un semblant d'algorithme (plus du tâtonnement d'ailleurs) qui fonctionne pour les petites valeurs de $N$ (dans $2^N$) mais je ne suis pas sûr qu'il aboutisse pour $2^{1453}$ (bataille de Castillon) ou $2^{1789}$. En outre, cela ne prouverait pas l'unicité.

    Cordialement, j__j
  • Est-ce qu'il est prévu de prolonger le calendrier de l'Avent par un calendrier de l'Arriare ? Ce fil est bien sympathique. 
  • Modifié (December 2022)

    Problème 25

    1) Une fonction et une propriété des itérés

    On pose $f(x)=\dfrac{4x-1}{4}\sqrt2+\dfrac{1}{4}$
    On a alors $f\circ f (x)=\dfrac{4x-1}{4}\sqrt2^2+\dfrac{1}{4}$ ;  $f^{-1}(x)=\dfrac{4x-1}{4}\sqrt2^{-1}+\dfrac{1}{4}$,
    et en général : $f^{a}(x)=\dfrac{4x-1}{4}\sqrt2^{a}+\dfrac{1}{4}$, pour tout $a \in Z$

    2) Une autre écriture de $u_{n,m}$
    On pose $i=4-k$, de sorte que $u_{n,m}=\displaystyle \sum _{i\leq n-1}\lfloor f^i(m)\rfloor$
    (à  suivre)
  • Modifié (December 2022)
    Problème 25 (suite et fin)

    3) Un classement
    Soit $h$ un entier tel que $1\leq h \leq f^{n-1}(m)$, alors $ f(h) \leq f^{n}(m)$ et $\lfloor f^{n-1}(m) \rfloor$ représente le nombre de $f(h)$ qui sont inférieurs ou égaux à $ f^{n}(m)$.
    De même $\lfloor f^{n-2}(m) \rfloor$ représente le nombre de $f^2(h)$ qui sont inférieurs ou égaux à $ f^{n}(m)$.
    Ainsi de suite.
    $u_{n,m}$ est donc le nombre des $f^i(j)$ qui sont inférieurs ou égaux à $ f^{n}(m)$.
    $u_{n,m}$ est le rang de $f^n(m)$ quand on classe les $f^a(b)$ par ordre croissant ($a\in N^*, \quad b\in N^*$).
    Nous avons ainsi une bijection de $(\N^*)^2$  sur $\N^*$.
  • Merci, Cidrolin ! Je n'étais pas du tout parti dans cette direction ; je cherchais plutôt du côté des réduites de la décomposition de $N/(2+\sqrt2)$ en fraction continue.
  • Modifié (December 2022)
    Math Coss a dit :
    Est-ce qu'il est prévu de prolonger le calendrier de l'Avent par un calendrier de l'Arriare ? Ce fil est bien sympathique. 
    C'était bien, mais pas sûr que le fil tienne le rythme d'une manière permanente. Il vaut peut être mieux ne pas abuser des bonnes choses, et attendre l'année prochaine pour remettre ça.
  • Belle construction collective ! Pour ceux qui sont enseignants et ne le savent pas, Mathématiques sans Frontières à mis en ligne un calendrier de l’avent pour un publique CM2 à 3e. Je l’ai suivi en classe et je peux vous dire qu’il est vraiment bien conçu pédagogiquement. Les élèves en redemandaient ! Le fond mathématiques est très pertinent et c’est l’occasion d’une cure de résolution de problèmes très salvatrice.
  • Math Coss a dit :
    Est-ce qu'il est prévu de prolonger le calendrier de l'Avent par un calendrier de l'Arriare ? 

    On peut remettre tous les jours l'exo non résolu de 2005 :)
  • @philou22 a dit :
    Pour ceux qui sont enseignants et ne le savent pas, [...]


    Tu crois qu'il y a beaucoup d'enseignants qui ne savent pas qu'ils sont enseignants ? :p


  • Je ne vous oublie pas, mais il y a encore du boulot de mise en forme et quelques trous à boucher.
  • bisam a dit :
    @philou22 a dit :
    Pour ceux qui sont enseignants et ne le savent pas, [...]


    Tu crois qu'il y a beaucoup d'enseignants qui ne savent pas qu'ils sont enseignants ? :p

    Je voulais dire "pour les enseignants qui ne sont pas encore au courant de l'existence du calendrier". Je reconnais que c'étais mal formulé !

  • Ou alors un exercice par semaine ? Peut-être ? Même s’il est résolu à vitesse Grand V, c’est en effet bien plaisant. Peut-être aussi permettre tous les niveaux, notamment avec des recommandations des auteurs (utiliser ceci, sans les congruences, etc.). 
  • @Dom, @Math Coss : Il y a déjà des exercices-défis régulièrement sur le forum (souvent proposés par etanche en fait). Éventuellement, nous pouvons faire quelques efforts pour en proposer plus.

    @rémi : pas de souci ! 
  • Exact. 
  • Modifié (January 2023)
    bd2017 a dit :

    Jour 17

    Montrer que pour tout $a,b,c \in \R$ on a $$ \sum_{cycl} (a^4 - a^3 b + 8 a^2 b^2 - 7 a^2 b c - a b^3 ) \geq 0.$$
    Il ne me semble pas que nous en ayons eu la solution ! J'ai tenté diverses approches : en particulier, montrer que cette application est convexe (mais la vérification me semble trop compliquée) et remarquer que l'on a un minimum local en les points de coordonnées de la forme $(a,a,a)$.

    À moins que, par chance, cette fonction ne soit un polynôme symétrique en les trois arguments et que l'on puisse de ce fait en donner une expression en les polynômes symétriques élémentaires ??
  • Dom a dit :
    Ou alors un exercice par semaine ? Peut-être ? Même s’il est résolu à vitesse Grand V, c’est en effet bien plaisant. Peut-être aussi permettre tous les niveaux, notamment avec des recommandations des auteurs (utiliser ceci, sans les congruences, etc.). 
    Ce peut être une bonne idée :)
    Je suis donc je pense 
  • Une autre idée possible pour J17 était : si l'on montre que $f(a,b,c)$ ne s'annule que sur la droite d'équation $X=Y=Z$, alors $f$ garde un signe constant puisque le complémentaire de cette droite est connexe par arcs. Je vais peut-être insister dans cette direction (mais il serait bon de trouver des zéros évidents de $f(X,b,c)$ par exemple.
  • Modifié (January 2023)
    Bonjour
    Merci @john_john d'avoir  regardé mon problème un peu oublié par tout le monde. Concernant ton idée que je découvre ici, pourquoi pas. 
    Ma solution est complètement différente. J'attends donc encore un peu pour la donner. De toute façon, pour le moment   il y a un  problème avec le forum, les mathématiques ne s'affichent pas.  
     
  • Première étape : le polynôme est totalement symétrique.
    sage: var('a b c')
    (a, b, c)
    sage: e1 = a^4 - a^3* b + 8 *a^2* b^2 - 7* a^2 *b* c - a* b^3 
    sage: D = {a: b, b: c, c: a}
    sage: e2 = e1.subs(D)
    sage: e3 = e2.subs(D)
    sage: e = e1+e2+e3
    sage: e.subs({a:b, b:a})-e
    0
    

  • Modifié (January 2023)
    (J17) Merci, MathCoss ! J'ai eu tort de penser que les calculs allaient être très laborieux par cette voie -- ce n'est pas pour autant que ce qui suit est de la dentelle. Alors, voici : 

    La fonction $f$ est symétrique et donc polynomiale en $s_1=a+b+c$, etc. La méthode des poids donne facilement $f=s_1^4-5s_1^2s_2+12s_2^2-18s_1s_3$.

    Si $s_1=0$, l'inégalité est acquise ; sinon, vu l'homogénéité, on peut se ramener à $s_1=3$ et poser $a=1+x,b=1+y,c=1+z$.

    En les nouvelles variables : $f=27\sigma_2+12\sigma_2^2-54\sigma_3$ ; or, $4\sigma_2^3+27\sigma_3^2\leqslant0$ (discriminant).

    Donc, $f\geqslant3(9\sigma_2+4\sigma_2^2-4\sqrt3|\sigma_2|^{3/2})$

    Comme $\sigma_2\leqslant0$, on peut poser $\sigma_2=-u^2$ et étudier le signe de $u^2(4u^2-4\sqrt3u+9)$ ; du nanan --- c'est bien positif.

    bd2017 peut-il nous dire si cette inégalité est miraculeuse ou si, comme je le suppose, il existe une fabrique de formules de ce type (je m'explique : la définie positivité de la matrice de Hilbert semble miraculeuse tant que l'on n'a pas remarqué que cela découle de l'inégalité $\displaystyle\int_0^1(\sum a_kx^k)^2{\rm d}x\geqslant0$, qui permet dès lors de fabriquer des exemples ad nauseam).


  • Remarque : on a en prime le cas d'égalité $f=0$ : dans le cas où $s_1\neq0$, cela se produit si, et seulement si, $\sigma_2=0$ et donc aussi $\sigma_3=0$. Tout cela équivaut à $a=b=c$.

    Dans le cas où $s_1=0$, une CNS est que $s_2=0$ et donc $s_3=0$ puisque l'équation $X^3-s_3=0$ a trois solutions réelles. On obtient là encore $a=b=c=0$.
  • J'ai mis un certain à comprendre pourquoi @john_john affirme deux messages au-dessus que $4\sigma_2^3+27\sigma_3^2\leq 0$.
    Des fois que ça pose aussi problème à d'autres: le discriminant de $(X-x)(X-y)(X-z)$ est $-4\sigma_2^3-27\sigma_3^2$ (puisqu'on suppose que $\sigma_1:=x+y+z=0$), et est positif puisqu'on suppose que le polynôme a trois racines réelles. Ceci implique en particulier que $\sigma_2\leq 0$. Bien vu.
    Après je bloque.
  • Bien vu, le changement de variable pour ramener à $x^3+px+q$ ! Hier soir, j'ai bien écrit l'expression $f=f=s_1^4-5s_1^2s_2+12s_2^2-18s_1s_3$ et l'expression générale du discriminant mais je ne savais plus du tout quoi faire après.
  • Modifié (January 2023)
    Bonjour
    Bravo @john_john
    Voici pour répondre peut être à ta question:  Voici  comment j'ai fabriqué l'exercice.
    Soit $A= \sum_{cycl} a^2(a-b)(a-c) $  (et on a $A\geq 0$, c'est l'inégalité de Schur)  et $B=   \sum_{cycl} a^2(b-c)^2 \geq 0 , \forall a,b,c .$
    Puis $f(a,b,c)=A + 4 B.$  Trouver cette décomposition est aussi une solution mais dans un certain sens ce n'est pas aussi évident.

    Sinon voici ma solution: $f$  est "cyclique"  ($f(a,b,c)=f(b,c,a) $  )    et "symétrique"    ($ f(a,b,c)=f(b,a,c)$)  , $\forall a,b,c.$
    On peut donc supposer $a\leq b\leq c$  et posons  $b=a+h,c=a+h+k, h, k\geq 0.$
    On a alors $$f(a,b,c)=9 a^2 h^2+9 a^2 h k+9 a^2 k^2+16 a h^3+24 a h^2 k+12 a h k^2+2 a k^3+8 h^4+16 h^3 k+11 h^2 k^2+3 h k^3+k^4=w(a)$$   
    et $w$ est un polynôme du second degré par rapport à la variable $a$. On calcule le discriminant
    $$w'(0)^2- 2 w''(0) w(0)= -32 h^6-96 h^5 k-300 h^4 k^2-440 h^3 k^3-300 h^2 k^4-96 h k^5-32 k^6 $$
    qui est manifestement négatif. D'où le résultat .



     
  • Modifié (January 2023)
    Comme je ne connais pas l'inégalité de Schur, j'ai voulu la démontrer avec cette méthode : cela fonctionne du tonnerre ! In:
    var('a b c')
    e1 = a^2*(a-b)*(a-c)
    D = {a: b, b: c, c: a}
    e2 = e1.subs(D)
    e3 = e2.subs(D)
    e = e1+e2+e3
    e.subs({a:b, b:a})-e
    Out:
    0
    In:
    var('h k')
    f = e.subs({b:a+h, c:a+h+k})
    QQ[h,k][a](f), QQ[h,k][a](f).discriminant().factor()
    Out:
    ((h^2 + h*k + k^2)*a^2 + (4*h*k^2 + 2*k^3)*a + 3*h^2*k^2 + 3*h*k^3 + k^4,
     (-12) * k^2 * h^2 * (h + k)^2)


  • Modifié (January 2023)
    i.zitoussi : effectivement, je n'ai pas expliqué l"inégalité invoquée car j'ai pensé que, avec le mot discriminant, tout le monde ferait, comme tu l'as fait, le rapprochement avec la réalité des racines d'une équation de degré $3$ sous forme canonique.

    Math Coss : le changement de variable était surtout un coup de chance, car j'avais la flemme de calculer le discriminant dans le cas général (ce qui m'aurait pris toutefois moins de temps que si j'avais dû me replonger dans le mode d'emploi de XCas).

    bd2017 : je suis content d'avoir appris l'inégalité de Schur ! Se généralise-t-elle avec plus de variables ? [Merci Wikipedia : ce que l'on peut généraliser, c'est l'exposant.]
    Amitiés à tous
  • Il est intéressant également de traiter le cas de $\sum a^2(a-b)(a-c)$ : on obtient in fine $u^2(2u-\sqrt3)^2\geqslant0$, ce qui donne deux cas d'égalité cette fois : $a=b=c$ et, à permtation près, $a=b, c=0$, ce qui confirme Wikipédia.
  • Modifié (January 2023)
    Bonjour
    Sauf erreur il n'y a pas d'inégalité de Schur avec 4 variables. Par exemple  avec  la somme cyclique $a^2 (a-b)(a-c)(a-d) $
    $a=b=c=1$ et $d=\pm 2$  donne deux signes différents.
     
Connectez-vous ou Inscrivez-vous pour répondre.
Success message!